Which shows the expression below simplified?
0.0042 - (1.2 x 10-6)

A. 3 x 10^-3
B. 1.1958 x 10^-6
C. 4.1988 x 10^-6
D. 4.1988 x 10^-3

Answers

Answer 1

The shows the expression below simplified 0.0042 - (1.2 x 10-6) as D. 4.1988 x 10^-3.

How can the expression be simplified?

The concept that is been used is subtraction of the first expression from second expression.

The given expression is 0.0042 - (1.2 x 10-6) and this can be expressed in standard form as ( 4.2 *10^-3) - (1.2 x 10-6)

Then then the subtraction can be done as [ ( 4.2 *10^-3) ] - [ (1.2 x 10^-6) ] = 4.1988 x 10^-3

In conclusion, iif we go through the options., then we can see that the forth option is the best option that match the expression.

Therefore, option D is correct.

Read more about expression at:

https://brainly.com/question/723406

#SPJ1


Related Questions

Please assist Math with 50 points!

Answers

Answer:

A. (1/2) bucket

Step-by-step explanation:

    6           11

4 ------ - 3 ------- = ?

    14          12

14 × 4 = 56

56 + 6 = 62

12 × -3 = -36

-36 - 11 = -47

 62       47

------- - -------

  14        12

 62(12)        47(14)

-------    -     -------

  14(12)         12(14)

744     658       86

------- - ------- = --------

 168     168       168

 86 ÷ 2

-------

168 ÷ 2

43

----- = 0.511

84

0.511 ≈ (1/2)

I hope this helps!

Answer:

1/2 bucket

Step-by-step explanation:

What is the value of logx?y3lodwhen given the following:Zlog(x) = 3log(y)=2log(z)= -1

Answers

Given the question

[tex]\log (\frac{x^2y^3}{z})[/tex]

To resolve this, we can follow the steps below

Step1: Apply the logarithm rule

[tex]\log (\frac{x^2y^3}{z})=\log x^2+\log y^3-\log z[/tex]

This will give=>

[tex]\log x^2+\log y^3-\log z=2\log x+3\log y-\log z[/tex]

Since we have been given that

[tex]\begin{gathered} \log x=3 \\ \log y=2 \\ \log z=-1 \end{gathered}[/tex]

Step2: Substitute the given values into the equation

[tex]2\log x+3\log y-\log z=2(3)+3(2)-(-1)[/tex]

=>

[tex]6+6+1=13[/tex]

Answer = 13

a potter forms a piece of clay into a right circular cylinder. as she rolls it, the height of the cylinder increases and the radius decreases. assume that no clay is lost in the process. suppose the height of the cylinder is increasing by centimeters per second. what is the rate at which the radius is changing when the radius is centimeters and the height is centimeters?

Answers

The radius of the cylinder is decreasing at the rate of 0.22 cm per second.

One of the most important and fundamental curvilinear of a geometric shapes, a right circular cylinder has historically been a three-dimensional solid.

It is regarded as a prism with a circle as its base in basic geometry. In several contemporary fields of geometry and topology, a cylinder can alternatively be characterized as an infinitely curved surface.The radius of the cylinder is the length of the line joining the circumference and the center of the circle.The derivative of the logarithm,  dx /dt, is also the growth rate. The fractional change, must be used to represent a minor change in the logarithm. if the variable grows at the constant fixed rate g.

Volume of a right circular cylinder = π r² h

Now we differentiate w.r.t to get :

[tex]\frac{dV}{dT} =2 \pi r h\frac{dr}{dt} + \pi r^2\frac{dh}{dt}[/tex]

Taking the values we get :

[tex]-34.3 = 154 \frac{dr}{dt}[/tex]

[tex]\frac{dr}{dt} = -0.22[/tex]

Hence the radius is decreasing at the rate of -0.22 cm per second.

To learn more about cylinder visit:

https://brainly.com/question/16134180

#SPJ4

What is the image point of (-7,3) after the transformation rx-axis o R180º?

Answers

[tex]r_{x-axis}\circ R_{180}[/tex]

In a composite transformation as given you make first the transformation in the right and then the transformation in the left.

For the given point: (-7, 3)

1. Rotation 180°

[tex]\begin{gathered} P(x,y)\rightarrow P^{\prime}(-x,-y) \\ \\ P(-7,3)\rightarrow P^{\prime}(7,-3) \end{gathered}[/tex]

2. Reflection over x-axis:

[tex]\begin{gathered} P^{\prime}(x,y)\rightarrow P^{\prime}^{\prime}(x,-y) \\ \\ P^{\prime}(7,-3)\rightarrow P^{\doubleprime}(7,3) \end{gathered}[/tex]Then, the image of given point after the composite transformation is (7,3)

Which inequality represents the graph?

y less than negative one-fourth (x minus 10) squared + 4
y greater than negative 4 (x + 10) squared + 4
y greater than one-fourth (x minus 10) squared + 4
y greater than 4 (x + 10) squared + 4

Answers

Answer: 1st option!

Step-by-step explanation:

Just did it on edg

48/8 6/w

1=?



Help me please helpp

Answers

Answer:

w = 1

Step-by-step explanation:

You have to divide 48 by 8 to get 6, so you have to divide 8 by 8 to find w.

8/8 = 1

Please helppp as fast as possible 20 points ***
FIND THE DOMAIN & RANGE OF THE FUNCTION
g(x)=|x + 4|

Please helppp as fast as possible

Answers

Answer:

Domain: (−∞,∞),{x | x ∈ R}

Range: [0,∞),{y|y≥0}

Step-by-step explanation:

PLEASE HELP!!!!!!!!! It’s algebra

Answers

Answer:

See below

Step-by-step explanation:

From 5 to 10 mugs is 5 mugs  

   These cost  110 - 67.50 = 42.50

        So each mug costs 42.50 / 5 = 8.50 each  

              With a 'base cost' of 25 dollars

   (Base cost might be artwork, design, order processing, shipping or whatever.....but this amount is added to each order)

(As a check    20 mugs would be   25 + 20 (8.50) = 195   <====yep

150 mugs will then be 25   +  150 ( 8.50) = 1300 dollars

write an equation of the line below.​

Answers

Answer:

y = 2x + 3

Step-by-step explanation:

The y-intercept is where x = 0. Therefore, the y-intercept would be 3. To find the slope, you do Δy/Δx (Δ = Change In). To find that, you go up 4 times, then move to the right 2 times to reach the second dot. Now, you do 4 / 2, which equals 2. Your final equation will be y = 2x + 3.

find the volume of the rectangular prism. need an integer or decimal.

Answers

We will have the following:

[tex]V=(\frac{9ft\ast9ft}{2})(12ft)\Rightarrow V=486ft^3[/tex]

So, the volume is 486 ft^3.

Given g(x)=−23(x−4)2+3, g ( x ) = − 2 3 ( x − 4 ) 2 + 3 , what is the value of g(–5)? *Type in your answer. g(–5) =

Answers

The output value of g( -5 ) in the function g( x ) = −23( x - 4 )2 + 3 is 417.

What is the output value of g(-5) in the given function?

A function is simply a relationship that maps one input to one output, that is, each x-value can only have one y-value.

Given the data in the question;

g( x ) = −23( x - 4 )2 + 3g( -5 ) = ?

To find the g( - 5 ), replace all the occurrence of x with -5 in the function and simplify.

g( x ) = −23( x - 4 )2 + 3

g( -5 ) = −23( -5 - 4 )2 + 3

g( -5 ) = −23( -9 )2 + 3

g( -5 ) = −23( -9 × 2) + 3

g( -5 ) = −23( -18) + 3

g( -5 ) = ( -23 × -18) + 3

g( -5 ) = ( 414 ) + 3

g( -5 ) = 414 + 3

g( -5 ) = 417

Therefore, the output value of g( -5 ) is 417.

Learn more about functions here: brainly.com/question/2541698

#SPJ1

HELP ME PLEOPLE THIS IS SO CONFUSING HELP ME I WILL GET SUSPENDED IF I DON"T ANSWER THIS HELP ME PLEASE HELP I WILL GET A 0 AND SUSPENDED HELP

Answers

D and B ! Your welcome!


How do we calculate slope when given two points on a coordinate plane?

Answers

Step-by-step explanation:

Given the coordinates of two points on a line, use the slope formula to find the slope of the line. The slope formula is m=(y2-y1)/(x2-x1) or the change in y value versus the change in x value. The coordinates of the first point represent x1 and y1. The coordinates of the second point are x2, y2.

Answer:

To find the slope, use the formula Rise over Run.

1.) Insert the Y coordinates into their correct space.

2.) Plug in the X coordinates.

3.) Subtract your Y coordinates.

4.) Subtract your X coordinates.

5.) Reduce if needed.

6.) You're done!

Solve using substitution.
X = 7
-2x 9y = 13
(__ , __)?
Submit

Answers

Answer:

(7,-3)

Step-by-step explanation:

x = 7

-2x - 9y = 13

* Substituition is the easieat way to solve such a function.

- Firstly, put "7" instead of "x" in the equation.

- multiply "7" by "-2"

- Solve for "y"

* -2x-9y = 13

* -2×7-9y = 13

* -14-9y = 13

* -9y = 27

* -9y/-9 = 27/-9

* y = -3

# Since x = 7, the output/"y" will be -3

(7,-3)

PLEASE ANSWER QUICK IT IS THE LAST PROBLEM ON HERE AND IT IS DUE TODAY!!!

Answer should be inequality!

Ray had scores of 75, 82, 94, and 77 on his first four algebra tests. What must he score on the next test to have an average of at least 85?

Please show work as well if possible.

Answers

Answer:

x ≥ 97

Step-by-step explanation:


Average = Sum of values divided by the amount of numbers in a data set


Data Set: {75, 82, 94, 77, ?}

75 + 82 + 94 + 77 + ? = ? ÷ 5 = 85

85 x 5 = 425

75 + 82 + 94 + 77 + ? = 425 ÷ 5 = 85

75 + 82 + 94 + 77 = 328


425 - 328 = 97

Ray must score greater than or equal to 97, or x ≥ 97 in order to have an average of at least 85.

Chad doesnt have any money, but he owes his sister $15, owes his brother $9 and owes his friend $24. What integer could be used to represent the amount of money chad has?

Answers

Answer:

- 48.00

Step-by-step explanation:

Add all the debt up.

Answer: -48

Step-by-Step Explanation:

Money that Chad owes his Sister = $15

Money that Chad owes his Brother = $9

Money that Chad owes his Friend = $24

Total Money he owes = 15 + 9 + 24 = $48

Hence, Total Money he has = -48 (since he doesn't have any money)

vAt her Fourth of July party, Sally serves her famous grilled chicken. Her secret is to use 6 fluid ounces of lemon-garlic marinade for every pound of chicken. Sally plans to grill 4 pounds of chicken. How many cups of marinade should she use

Answers

In linear equation, 3 number of cups will be needed to marinade 4 pounds of chicken for grill.

What are a definition and an example of a linear equation?

An equation with only one variable is referred to as a linear equation in one variable. It has the mathematical formula Ax + B = 0, where A and B can be any two real numbers, and x is an unknowable variable with just one possible value. A linear equation in one variable would be 9x + 78 = 18, for instance.

Here we have the data from the question:-

Marcy will use 6 fluid ounces of lemon-garlic marinade for every pound of chicken.

Marcy plans to grill 4 pounds of chicken.

So total marinade required will be:-

Total marinade  =6x4=24 ounces

Now we know that the conversion:-

1 fluid Ounce = 0.125 Cup

24 fluid ounce =0.125 Cup

Hence 3 number of cups will be needed to marinade 4 pounds of chicken for grill.

Learn more about linear equation

brainly.com/question/11897796

#SPJ10

Hi I just need Part A. I’m in high school calculus 1 and this is a homework. thank you !

Answers

SOLUTION

Given the question in the image, the following are the solution steps to answer the question.

STEP 1: Define what happens when a function is shrunk vertically

When the factor is greater than 1, the condition that satisfies the function is to multiply by 1.4

When the graph is verticcally shrunk, this means that the function is multiplied by 1.4

Hence, the function becomes:

[tex]1.4\times f(x)=1.4\cdot f(x)[/tex]

STEP 2: Define what happens when the graph is shifted left by 3 units.

If the function is shifted left by 3 units, this implies that we add 3 units to the x of the function.

This gives:

[tex]1.4.f(x+3)[/tex]

Solve the inequality and graph the solution on the line provided.
3x+17_< 41

Answers

So if your problem is: 3x + 17 (less than or equal to sign) 41

Your answer should be: x _< 8

(The “<“ is on top of the underline btw)

The sign between x & 8, should be: “less than or equal to” sign.

And your point on the graph should go form right to left. Starting on right side “8”, then proceeding to the negative side

Check the graph below that I provided

The equation for line s can be written as x - y = -2. Line t is perpendicular to lines and passes through (-4, 1). What is the equation of line t?​

Answers

Answer:  x + y = -3

=========================================================

Explanation:

Anything perpendicular to Ax+By = C is of the form Bx-Ay = D

The given equation is x-y = -2 showing that A = 1, B = -1 and C = -2.

So Bx-Ay = D will update to -x-y = D

Plug in the coordinates of (-4,1) to find D.

-x-y = D

D = -x-y

D = -(-4)-1

D = 4-1

D = 3

We go from -x-y = D to -x - y = 3

Then multiply both sides by -1 to end up with x + y = -3

I have a graph with the question. If the parent function is y= 2 to the power of x plus 2, which is the function of the graph

Answers

The parent function of the graph is

[tex]y=2^x[/tex]

The function of the grapg will be

[tex]y=2^{x\text{ + a}}\text{ + b}[/tex]

at x equals infinity y equals 1

Therefore

[tex]\begin{gathered} \text{x =}\infty\text{ then y = 1} \\ \text{this implies} \\ b\text{ = 1} \end{gathered}[/tex]

Therefore the function will now be

[tex]y=2^{x\text{ + a}}\text{ + 1}[/tex]

from the graph

x = 2 when y = 2

Therefore,

[tex]\begin{gathered} 2=2^{2\text{ + a}}\text{ + 1} \\ 2-1=2^{2\text{ + a}} \\ 1=2^{2\text{ + a}} \\ \text{recall 1 = 2}^0 \\ 2^0=2^{2\text{ + a}} \\ 2\text{ + a = 0} \\ a\text{ = - 2} \end{gathered}[/tex]

Therefore, the function is

[tex]y=2^{x\text{ - 2}}\text{ + 1}[/tex]

someone include steps if you can

Answers

Using the concept of the equation, it can be concluded that Levi answered the first question wrong and the last two correctly. Thus,

No

Yes

Yes

How to check whether the value of x satisfies the equation?

To check the value of x satisfies the equation, put the value of x and if it matches the resultant, it is correct.

Now solving the first part, x/4=20;x=5

So, putting x=5 in the equation 5/4=1.25 is not matching with the resultant ie. 20.

Therefore, this is the incorrect value of x.

Now solving the second part, 1.5x-8=7;x=10

So, putting x=10 in equation 1.5*10-8= 15-8=7

It matches with the resultant.

Therefore, this is the correct value of x.

Now solving the third part, 4x+6=58; x=13

So, putting x=13 in equation 4(13)+6= 52+6=58

It matches with the resultant.

Therefore, this is the correct value of x.

To know more about equations, visit:

https://brainly.com/question/10413253

#SPJ13

The larger of two numbers is five times the difference of the small number and one. If sum of the two numbers is 50 what are the two numbers?

Answers

Answer:

Step-by-step explanation:

40.833 recurring and 9.166 recurring

or 40 5/6 and 9 1/6

Call the small number 's'

Call the big number 'b'

The word problem says:

B+S=50 and also that 5(S-1)=B which multiplied out is 5S-5=B

So, 5S-5=B=50-S

Therefore 6S=55 and the rest history!

Answer:

Step-by-step explanation:

si cada 8.33 minutos me dan 10 dolares cuantos dolares tendria en 6 horas ademas cuantos dolares ganaria cada 30 minutos

Answers

The payment for each 6 h is equal to $ 432.2 while the payment for each 30 min would be  $ 36.01.

Rule of Three

It's a math tool for solving problems about proportion. You can relate more than 2 variables and from the proportion, it's possible to find an unknown number.

Conversion Unit Time

In the International System Units (SI), the standard unit for time is the second (s).  For solving any exercise, you need to know the relation between the units of the time. See below:

1h=60 min = 3600s

1 min = 60 s

Now you can solve the question from the rule of three for finding the payment in dollars.

The question gives: an each 8.33 min, the payment is 10 dollars. Thus, from this relation you can write:

For 6 h

First, you should convert 6h to minutes, then,

1 h ----- 60 min

6h ----- x

x=60*6=360 min

After that, you should apply the rule of three for finding the payment in dollars.

   8.33 min     -----  10 dollars.

   360 min    -------  x

8.33x=360*10

8.33x=3600

x= $ 432.2

For 30min

You should apply the rule of three for finding the payment in dollars.

   8.33 min     -----  10 dollars.

   30 min    -------  x

8.33x=30*10

8.33x=300

x= $ 36.01

Read more about the rule of three here:

brainly.com/question/24514897

#SPJ1

Find the area of 2x-9 + x+5

Answers

=3x - 4
Hope this helps :)

Answer: 3x-4

Step-by-step explanation: First Simplify, add the numbers 2x-9+x+5=2x-4+x Then combine like terms, 2x-4+x= 3x-4

can you explain each step please for i can write it on a paper along with you

Answers

The two groups are given one is those who are studied and other is those who are not studied.

Studied - 88,100,94,79,92,100,95,83,89,99,100,91,89,95,100,93,96,84.

Arrange in order.

79, 83, 84, 88, 89, 89, 91, 92, 93, 94, 95, 95, 96, 99, 100, 100, 100, 100

The mean of this group is

[tex]\frac{88+100+94+79+92+100+95+83+89+99+100+91+89+95+100+93+96+84}{18}[/tex][tex]\frac{1667}{18}=92.6[/tex]

The mean is 92.6.

The median of this group is

Use the median formula for even

[tex]m=\frac{(\frac{n}{2})^{th}+(\frac{n}{2}+1)^{th}^{}}{2}[/tex]

Substitute the value of n =18

[tex]m=\frac{(\frac{18}{2})^{th}+(\frac{18}{2}+1)^{th}^{}}{2}=\frac{9^{th}+10^{th}^{}}{2}[/tex][tex]m=\frac{93+94}{2}=93.5[/tex]

The median is 93.5.

The mode of this group is 100 as it is appears 3 times .

The another group is not studied group.

Not studied - 82,72,45,91,58,83,65,87,90,77,73,89.

Arrange in order-

45, 58, 65, 72, 73, 77, 82, 83, 87, 89, 90, 91

The mean is determined as

[tex]\frac{82+72+45+91+58+83+65+87+90+77+73+89}{12}=\frac{912}{12}=76[/tex]

The median is determined as

[tex]m=\frac{(\frac{n}{2})^{th}+(\frac{n}{2}+1)^{th}}{2}[/tex]

Substitute n=12.

[tex]m=\frac{(\frac{12}{2})^{th}+(\frac{12}{2}+1)^{th}^{}}{2}=\frac{6^{th}+7^{th}}{2}=\frac{77+82}{2}=79.5[/tex]

There is no mode.

So, from the all the data given.

The mean of the group that studied is over 15 percent points higher than the mean of the group that did not studied.

[tex]92.61-76=16.61[/tex]

The median of the group that did not studied is less than 80.

In general , those students that studied scored much higher than those students that did not study.

The correct options are a , c and e.

If ax-3=bx+5 then the solution for x in terms of a and b is

Answers

In this equation the solution for x in terms of a and b is 8.

What is equation?

There are many different ways to define an equation. The definition of an equation in algebra is a mathematical statement that demonstrates the equality of two mathematical expressions. For instance, the equation 3x + 5 = 14 consists of the two expressions 3x + 5 and 14, which are separated by the 'equal' sign. Mathematical algebraic equations typically have one or more variables.

Sol- As per the question given that- ax-3= bx+5------(equation 1)

Let ax=3-----(equation 2)

And bx =5------(equation 3)

On adding equation(1)and (2) we get

(ax)+(bx)= 3+5

(ax)+(bx)=8

x(a+b)+y(a+b)=8

(x+y)+(a+b)=8

1×(a+b)=8

Thus

a+b=8

To know more about equation click -

https://brainly.com/question/2972832

#SPJ13

Choose a relationship model that will reach $0 in the same number of days as this scenario: mika has $12 and spends $2 each day. y = â€""2x â€"" 12 y = â€""3x 18 y = â€""4x 12 y = â€""12x 2

Answers

The equation that represent the relationship model that will reach $0 in the same number of days as the scenario gave is: y = -2x + 12

To solve this problem, we have to state the equation using the information of the problem.

Information about the problem:

Mika has = $12Mika spend = $2 per dayDay = xy = $0

Writing the equation according to the information gave, we have:

y = -2x + 12

Where:

y = 0$x = days

So that (y) equals zero, the number of days (x) should be = 6

y = -2x + 12

0 = -2*6 + 12

0 = -12 + 12

0 = 0

The "y" variable is the dependent variable, because it will change according to the number of days passed (x).

The "x" variable is the independent variable, because it doesn't depend on any other value, just the number of days passed.

What is an equation?

An equation is the equality between two algebraic expressions, which have at least one unknown or variable.

Learn more about equation at: brainly.com/question/2972832 and brainly.com/question/27815607

#SPJ4

this is a little hard for me help me out please

Answers

The simplified form of the expression as a fraction is -43/390.

Solving expressions and simplifying fractions

Fractions are expression written as a ratio of two integers. For instance, a/b are known as fractions.

Given the expression below:

[2-|-2/3-2(-1/5)|]÷(-13)

In order to solve the expression, we will use the PEMDAS rule to have:

[2-|-2/3-2(-1/5)|]÷(-13)

[2-|-2/3+1/10)|]÷(-13)

Find the LCM

[2-|-20+3/30|]÷(-13)

[2-|-17/30|]÷(-13)

[2-(17/30)]÷(-13)

[60-17/30]÷(-13)

43/30÷(-13)

Change the division sign to multiplication

43/30÷(-13)

43/30 * -1/13

-43/390

This gives the simplified form of the expression.

Learn more on simplifying expressions here: https://brainly.com/question/28036586

#SPJ1

The roots of the quadratic function are -2 and -6.Which of the following are the two factors of the quadratic expression?X + 2X-2X - 6X + 6X + 8X - 8

Answers

roots = {-2, -6}

Factors: (x + 6) and (x + 2)

Other Questions
Which postulate proves the two triangles congruent?A. SSSB. SASC. ASAD. AAS question content area prior to the adjusting process, accrued revenue has a.been earned and cash received b.not been recorded as revenue but cash has been received c.been earned and not recorded as revenue d.not been earned but recorded as revenue 3. Find the equation of a line passing through (5,-6) parallel to : x + 3y = 8 the points E,F,G and H all lie on the same line segment, in that order, such that ratio of EG:FG:GH is equal to 4:1:5. If EH=10, find EG Which statement describes how the reasonable domaincompares to the mathematical domain? 1. 2 x 5b 2. (3a) x 2 (b ) 3. 15d divided by 3d Complete the statement below using the drop-down menus.It is believed mitochondria evolved from ,while chloroplasts evolved from 50 POINTSMatch the descriptions with the correct person or event.----- was the sacred land of the Sioux and illegally occupied by the United States; also the site of the Great Sioux Indian War, including the massacre at Little Bighorn was the Nez Perce leader who declared that he would fight no more was bayoneted while resisting incarceration lost the battle of Little Bighorn was the site of the last armed Native American resistance was the site of the Sioux massacre of George Custer's 7th Cavalry----Wounded KneeBozeman TrailLittle BighornJohn ChivingtonCrazy HorseChief JosephGeorge A. CusterPromontory PointAndrew CarnegieThe Black Hills Question 7(Multiple Choice Worth 3 points)(05.04 LC)triangle PQR with side p across from angle P, side q across from angle Q, and side r across from angle RIf R measures 18, q equals 9.5, and p equals 6.0, then which length can be found using the Law of Cosines? p q RQ PQ problems created by differences between countries' antitrust rules, labor laws, patents, copyrights, taxes, product liability, and child labor laws relate to which forces in the global business environment? north safety products manufactures butyl gloves that offer permeation resistance to gas or water vapor for workers who use dangerous chemicals like ketones. the company has fixed costs of $12 million for its butyl glove production and unit variable costs of $3 per pair. if the company charges $7 per pair, how many pairs of gloves must it sell to break even? multiple choice 120,000 pairs 400,000 pairs 666,667 pairs 1,000,000 pairs 3,000,000 pairs A car is traveling at a speed of 70 kilometers per hour. What is the car's speed in miles per hour? How many miles will the car travel in 5 hours? In your computations, assume that 1 mile is equal to 1.6 kilometers. Do not round your answers. You and nine peers have been selected to apply for an important college scholarship. Only one student will be awarded the scholarship. You have been asked to write a brief essay to the scholarship committee in which you share an important event in your life that significantly changed you, your view about life, your life goals, or your character for the better. Apply narrative techniques, a solid prewriting strategy, and creative sentence patterns to share your story and stand apart from other applicants. what role do coalitions play in the policymaking process? choose 1 answer: choose 1 answer: (choice a) a politicians create coalitions to enact policy that the president supports (choice b) b politicians create coalitions to enact policy that has broad support from the public (choice c) c politicians create coalitions to enact policy only when they are not in the minority party (choice d) d politicians create coalitions to enact policy that appeals to their base given: department a 8,000 sq. ft., department b 5,000 sq. ft., and department c 6,000 sq. ft. the percent of overhead expense applied to department c to the nearest whole percent will be: Which excerpts from Act III of Hamlet show that plot events have resulted in Claudius feeling guilty? find the minimum value of the function f(x)=2x2-22x+68 to the nearest hundredth Which of these sentences is the best example of using sensory details to help the reader visualize the tsunami?a.A tsunami is a frightening experience for everyone involved. Not only is the sound of it terrifying, but the sight of all that water is also horrifying.c.If you are lucky, you see the tsunami before you hear it; a solid mass of inky ocean builds up into a wall out at sea. It sucks greedily at the water at the shoreline, leaving silvery fish flapping weakly in shallow pools.b.The sight of all that water building up and crashing is horrifying. It causes immense damage to both people and buildings.d.all of thesePlease select the best answer from the choices providedABCD What system did the Europeans established in Africa Compute, in centimeters and in meters, the height of a basketball player who is 6 ft 1 in. tall into cm and m